Med Surg Exam 5

Réussis tes devoirs et examens dès maintenant avec Quizwiz!

*A nurse is providing discharge teaching for a client who has a prescription for furosemide 40 mg PO daily. The nurse should instruct the client to take this medication at which of the following times of the day?* A. Morning B. Immediately after lunch C. Immediately before dinner D. Bedtime

A

*A patient who has brochiectasis asks the nurse, "What conditions would warrant a call to the clinic?"* a. Blood clots in the sputum b. Sticky sputum on a hot day c. Increased shortness of breath after eating a large meal d. Production of large amounts of sputum on a daily basis

A

*A patient with newly discovered high BP has an average reading of 158/98 mm Hg after 3 months of exercise and diet modifications. Which management strategy will be a priority for this patient?* a. Drug therapy will be needed because the BP is still not at goal. b. BP monitoring should continue for another 3 months to confirm a diagnosis of hypertension. c. Lifestyle changes are less important, since they were not effective, and medications will be started. d. More vigorous changes in the patient's lifestyle are needed for a longer time before starting medications.

A

*Which statement indicates the patient with asthma requires further teaching about self-care?* A. "I use my corticosteroid inhaler when I feel short of breath" B. "I get a flu shot every year and see my health care provider if I have an upper respiratory infection" C. "I use my inhaler before I visit my aunt who has a cat, but I only visit for a few minutes because of my allergies" D. "I walk for 30 minutes every day but sometimes I have to use my bronchodilator inhaler before walking to prevent me from getting short of breath"

A

B.C is diagnosed with thrombocytopenia. To evaluate the effectiveness of her treatment, the nurse plans to monitor the a. platelet count. b. prothrombin time. c. partial thromboplastin time. d. international normalized ratio (INR).

A

During treatment of a patient with a BP of 222/148 mm Hg and confusion, nausea, and vomiting, the nurse initially titrates the medications to achieve which goal? a. Decrease the mean arterial pressure (MAP) to 129 mm Hg b. Lower the BP to the patient's normal within the second to third hour c. Decrease the SBP to 160 mm Hg and the DBP to between 100 and 110 mm Hg as quickly as possible d. Reduce the systolic BP (SBP) to 158 mm Hg and the diastolic BP (DBP) to 90 mm Hg within the first 2 hours.

A

How does myelodysplastic syndrome (MDS) differ from acute leukemias? a. MDS has a slower disease progression. b. MDS does not result in bone marrow failure. c. MDS is a clonal disorder of hematopoietic ells. d. MDS affects only the production and function of platelets and WBCs.

A

The nurse determines that teaching about pernicious anemia has been effective when the patient says a. "This condition can kill me unless I take injections of the vitamin for the rest of my life." b. "My symptoms can be completely reversed if I take cobalamin (vitamin B12) supplements." c. "If my anemia does not respond to cobalamin therapy, my only other alternative is a bone marrow transplant." d. "The least expensive and most convenient treatment of pernicious anemia is to use a diet with foods high in cobalamin."

A

The nurse reviews the arterial blood gases of a patient. Which result would indicate the patient has later stage COPD? a. pH 7.32, PaCO2 58 mm Hg, PaO2 60 mm Hg, HCO3 30 mEq/L b. pH 7.30, PaCO2 45 mm Hg, PaO2 55 mm Hg, HCO3 18 mEq/L c. pH 7.40, PaCO2 40 mm Hg, PaO2 70 mm Hg, HCO3 25 mEq/L d. pH 7.52, PaCO2 30 mm Hg, PaO2 80 mm Hg, HCO3 35 mEq/L

A

What early manifestations is the patient with primary hypertension likely to report? A. No symptoms B. Cardiac palpitations C. Dyspnea on exertion D. Dizziness and vertigo

A

What is the most important method for identifying the presence of infection in a neutropenic patient? a. Frequent temperature monitoring b. Routine blood and sputum cultures c. Assessing for redness and swelling d. Monitoring white blood cell (WBC) count

A

What should the nurse emphasize when teaching a patient who is newly prescribed clonidine (Catapres)? a. The drug should never be stopped abruptly. b. The drug should be taken early in the day to prevent nocturia c. The first dose should be taken when the patient is in bed for the night d. Because aspirin will decrease the drug's effectiveness, Tylenol should be used instead

A

While performing blood pressure screening at a health fair, the nurse counsels which person as having the greatest risk for developing hypertension? a. A 56-year-old man whose father died at age 62 from a stroke b. A 30-year-old female advertising agent who is unmarried and lives alone c. A 68-year-old man who uses herbal remedies to treat his enlarged prostate gland d. A 43-year-old man who travels extensively with his job and exercises only on weekends

A

While receiving a unit of packed RBCs, the patient develops chills and a temperature of 102.2°F (39°C). What is the priority action for the nurse to take? a. Stop the transfusion and instill normal saline. b. Notify the health care provider and the blood bank. c. Add a leukocyte reduction filter to the blood administration set. d. Recognize this as a mild allergic transfusion reaction and slow the transfusion.

A

a patient is scheduled for exercises nuclear imaging stress testing. the nurse explains to the patient that this test involves a. IV administration of a radioisotope at the maximum heart rate during exercise to identify the heart's response to physical stress b. placement of electrodes inside the right-sided heart chambers through a vein to record the electrical activity of the heart directly c. exercising on a treadmill or stationary bicycle with continuous ECG monitoring to detect ischemic changes in the heart during exercise d. placement of a small transducer in four positions on the chest to record the direction and flow of the blood through the heart by the reflection of sound waves

A

*A nurse is reviewing the health record of a client who asks about using propranolol to treat hypertension. The nurse should recognize which of the following conditions is a contraindication for taking propranolol?* A. Asthma B. Glaucoma C. Hypertension D. Tachycardia

A Propranolol is a nonselective beta-adrenergic blocker. These recptors in the lungs causes bronchoconstriction.

*A nurse is transfusing a unit of packed red blood cells (PRBCs) for a client who has anemia due to chemotherapy. The client reports a sudden headache and chills. The client's temperature is 2° F higher than her baseline. In addition to notifying the provider, which of the following actions should the nurse take?* Select all that apply. A. Stop the transfusion. B. Place the client in an upright position with feet down. C. Remove the blood bag and tubing from the IV catheter. D. Obtain a urine specimen. E. Infuse dextrose 5% in water through the IV.

A The nurse should stop the transfusion for a rise in temperature of 2° F and reports of chills and fever. The client can be having a hemolytic reaction to the blood or a febrile reaction. C The nurse should avoid infusing more PRBCs into the client's vein, and should remove the blood bag and tubing from the client's IV catheter. D Obtaining a urine specimen to check for hemolysis is standard procedure when the client has a reaction to a blood transfusion.

*A nurse is teaching a client who has a new prescription for verapamil to control hypertention. Which of the following instructions should the nurse include?* A. Increase the amount of dietary fiber in the diet B. Drink grapefruit juice daily to increase vitamin C intake. C. Decrease the amount of calcium in the diet D. Withhold food for 1hr after the medication is taken

A increasing dietary fiber intake can help prevent constipation.

*When caring for a patient with thrombocytopenia, the nurse instructs the patient to* a. dab his or her nose instead of blowing b. be careful when shaving with a safety razor c. continue with physical activities to stimulate thrombopoiesis d. avoid Aspirin (ASA) because it may mask the fever that occurs with thrombocytopenia

A - Patients with thrombocytopenia should avoid aspirin because it reduces platelet adhesiveness, which contributes to bleeding. Patients should not perform vigorous exercise or lift weights. If a patient is weak and at risk for falling, supervise the patient when he or she is out of bed. Blowing the nose forcefully should be avoided. The patient should gently pat the nose with a tissue if needed. Instruct patients not to shave with a blade; an electric razor should be used.

*A complication of the hyperviscosity of polycythemia is* a. thrombosis b. cardiomyopathy c. pulmonary edema d. disseminated intravascular coagulation (DIC)

A - The patient with polycythemia may experience angina, heart failure, intermittent claudication, and thrombophlebitis, which may be complicated by embolization. These manifestations are caused by blood vessel distention, impaired blood flow, circulatory stasis, thrombosis, and tissue hypoxia, caused by the hypervolemia and hyperviscosity. The most common serious acute complication is stroke, caused by thrombosis.

*The nursing management of a patient in sickle cell crisis incudes* Select all that apply a. monitoring CBC b. optimal pain management and O2 therapy c. blood transfusions if needed and iron chelation d. rest as needed and deep vein thrombosis prophylaxis e. administration of IV iron and diet high in iron content

A, B, C, D - Complete blood count (CBC) is monitored. Infections are common with elevated white blood cell counts, and anemia may occur with low hemoglobin levels and low RBC counts. Oxygen may be administered to treat hypoxia and control sickling. Rest may be instituted to reduce metabolic requirements, and prophylaxis for deep vein thrombosis (with anticoagulants) is prescribed. Transfusion therapy is indicated when an aplastic crisis occurs. Patients may require iron chelation therapy to reduce transfusion-produced iron overload. Pain occurring during an acute crisis is usually undertreated; patients should have optimal pain control with opioid analgesics, nonsteroidal antiinflammatory agents, antineuropathic pain medications, local anesthetics, or nerve blocks.

*Priority nursing actions when caring for a hospitalized patient with a new onset temperature of 102.2 F and severe neutropenia include* Select all that apply a. starting the prescribed antibiotic STAT b. drawing peripheral and central line blood cultures c. ongoing monitoring of the patient's vital signs for septic shock d. taking a full set of vital signs and notifying the physician immediately e. administering transfusions of WBCs treated to decrease immunogenicity

A, B, C, D - Early identification of an infective organism is a priority, and cultures should be obtained from various sites. Serial blood cultures (at least two) or one from a peripheral site and one from a venous access device should be obtained promptly. In a febrile neutropenic patient, antibiotics should be started immediately (within 1 hour). Cultures of the nose, throat, sputum, urine, stool, obvious lesions, and blood may be indicated. Ongoing febrile episodes or a change in the patient's assessment findings (or vital signs) necessitates a call to the physician for additional cultures, diagnostic tests, addition of antimicrobial therapies, or a combination of these.

*Nursing interventions for a patient with severe anemia related to peptic ulcer disease include* Select all that apply a. instructions for high-iron diet b. taking vital signs every 8 hours c. monitoring stools for occult blood d. teaching self-injection of erythropoietin e. administration of cobalamin (vitamin B12) injections

A, C - Stool guaiac test is performed to determine the cause of iron-deficiency anemia that is related to gastrointestinal bleeding. Iron is increased in the diet. Teach the patient which foods are good sources of iron. If nutrition is already adequate, increasing iron intake by dietary means may not be practical. The patient with iron deficiency related to acute blood loss may require a transfusion of packed red blood cells (RBCs).

*Which findings indicate that a patient is developing status asthmaticus?* Select all that apply A. PEFR <300 L/min B. Positive sputum culture C. Unable to speak in complete sentences D. Lack of response to conventional treatment E. Chest x-ray shows hyperinflated lungs and a flattened diaphragm

A, C, D

*A patient is concerned that he may have asthma. Of the symptoms that he relates to the nurse, which ones suggest asthma or risk factors for asthma?* Select all that apply A. Allergic rhinitis B. Prolonged inhalation C. History of skin allergies D. Cough, especially at night E. Gastric reflux or heartburn

A, C, D, E

*Which treatments would the nurse expect to implement in the management plan of a patient with cystic fibrosis?* Select all that apply a. Sperm banking b. IV corticosteroids c. Airway clearance techniques (e.g., Acapella) d. GoLYTELY given as needed for severe constipation e. Inhaled tobramycin to combat Pseudomonas infection

A, C, D, E

*Which BP-regulating mechanism(s) can result in the development of hypertension if defective?* Select all that apply a. Release of norepinephrine b. Secretion of prostaglandins c. Stimulation of the sympathetic nervous system d. Stimulation of the parasympathetic nervous system e. Activation of the renin-angiotensin-aldosterone system

A, C, E

*A plan of care for the patient with COPD should include?* Select all that apply. A. Exercise such as walking B. High flow rate of O2 administration C. Low-dose chronic oral corticosteroid therapy D. Use of peak flow meter to monitor the progression of COPD E. Breathing exercises such as pursed lip breathing that focus on exhalation

A, E

*A patient is admitted to the hospital in hypertensive emergency (BP 244/142 mm Hg). Sodium nitroprusside is started to treat the elevated BP. Which management strategies would be most appropriate for this patient* Select all that apply a. Measuring hourly urine output b. Continuous BP monitoring with an arterial line c. Decreasing the MAP by 50% within the first hour d. Maintaining bed rest and providing tranquilizers to lower the BP e. Assessing the patient for signs and symptoms of heart failure and changes in mental status

A,B,E

*A nurse is assessing a client who has chronic neutropenia and who has been receiving filgrastim. Which of the following actions should the nurse take to assess for an adverse effect of filgrastim?* A. Assess for bone pain. B. Assess for right lower quadrant pain. C. Auscultate for crackles in the bases of the lungs. D. Auscultate the chest to listen for a heart murmur.

A. Bone pain is a dose-related adverse effect of filgrastim. It can be treated with acetaminophen and, if necessary, an opioid analgesic.

*A nurse is preparing to administer a transfusion of 300 mL of pooled platelets for a client who has severe thrombocytopenia. The nurse should plan to administer the transfusion over which of the following time frames?* A. Within 30 min/unit B. Within 60 min/unit C. Within 2 hr/unit D. Within 4 hr/unit

A. Platelets are fragile and should be administered quickly to reduce the risk of clumping. The nurse should administer the platelets within 15 to 30 min/unit.

*A nurse is teaching a client who has a new prescription for beclomethasone. Which of the following instructions should the nurse include?* A. "Rinse your mouth after each use of this medication." B. "Limit fluid intake while taking this medication." C. "Increase your intake of vitamin B12 while taking this medication." D. "You can take the medication as needed."

A. "Rinse your mouth after each use of this medication."

*A nurse is preparing to administer a dose of a new prescription of prednisone to a client who has COPD. The nurse should monitor for which of the following adverse effects of this medication?* Select all that apply A. Hypokalemia B. Tachycardia C. Fluid retention D. Nausea E. Black, tarry stools

A. CORRECT: The nurse should observe for hypokalemia. This is an adverse effect of prednisone. C. CORRECT: The nurse should observe for fluid retention. This is an adverse effect of prednisone. E. CORRECT: The nurse should monitor for black, tarry stools. This is an adverse effect of prednisone.

*A nurse is discharging a client who has COPD. The client is concerned about not being able to leave his house due to the need for staying on continuous oxygen. Which of the following responses should the nurse make?* A. "There are portable oxygen delivery systems that you can take with you." B. "When you go out, you can remove the oxygen and then reapply it when you get home." C. "You probably will not be able to go out as much as you used to." D. "Home health services will come to you so you will not need to get out."

A. CORRECT: The nurse should inform the client that there are portable oxygen systems that he can use to leave the house. This should alleviate the client's anxiety.

*A nurse at a provider's office is reviewing the laboratory test results for a group of clients. The nurse should identify that which of the following results indicates the client is at risk for heart disease?* Select all that apply A. Cholesterol (total) 245 mg/dL B. HDL 90 mg/dL C. LDL 140 mg/dL D. Triglycerides 125 mg/dL E. Troponin I 0.02 ng/mL

A. Cholesterol (total) 245 mg/dL C. LDL 140 mg/dL

*A nurse is providing information to client who has a new prescription for hydrochlorothiazide. Which of the following information should the nurse include?* A. Take the medication with food B. Plan to take the medication at bedtime. C. Expect increased swelling of the ankles D. Fluid intake should be limited in the morning.

A. Take the medication with food

*A nurse planning care for a client who has a PICC line in the right arm. Which of the following interventions should the nurse include?* Select all that apply A. Use a 10 mL syringe to flush the PICC line. B. Apply gentle force if resistance is met during injection. C. Cleanse ports with alcohol for 15 seconds prior to use. D. Maintain a transparent dressing over the insertion site. E. Flush with 10 mL heparin before and after medication administration.

A. Use a 10 mL syringe to flush the PICC line. C. Cleanse ports with alcohol for 15 seconds prior to use. D. Maintain a transparent dressing over the insertion site.

*A nurse is teaching a client who has a prescription for long-term use of oral prednisone for treatment of chronic asthma. The nurse should instruct the client to monitor for which of the following manifestations as an adverse effect of this medication?* A. Weight gain B. Nervousness C. Bradycardia D. Constipation

A. Weight gain

*A nurse is providing teaching for a client who has a new diagnosis of hypertension and a new prescription for spironolactone 25 mg/day. Which of the following statements by the client indicates an understanding of the teaching?* A. "I should eat a lot of fruits and vegetables, especially bananas and potatoes. " B. "I will report any changes in heart rate to my provider. " C. "I should replace the salt shaker on my table with a salt substitute. " D. "I will decrease the dose of this medication when I no longer have headaches and facial redness. "

B

*While obtaining subjective assessment data from a patient with hypertension, the nurse recognizes that a modifiable risk factor for the development of hypertension is* a. A low-calcium diet b. Excess alcohol intake c. A family history of hypertension d. Consumption of a high-protein diet

B

A 45-year-old patient has symptoms including arthralgia, impotence, weight loss, and liver enlargement. His laboratory results include an elevated serum iron, total iron binding capacity (TIBC), and serum ferritin levels. Which disorder does this describe and which treatment will be used? a. Thalassemia; combination chemotherapy b. Hemochromatosis; deferoxamine (Desferal) c. Myelodysplastic syndrome; filgrastim (Neupogen) d. Delayed transfusion reaction; deferasirox (Exjade)

B

A patient has a platelet count of 50,000/μL and is diagnosed with ITP. What does the nurse anticipate that initial treatment will include? a. Splenectomy b. Corticosteroids c. Administration of platelets d. Immunosuppressive therapy

B

A patient with acute myelogenous leukemia is starting chemotherapy. When teaching the patient about the induction stage of chemotherapy, what is an appropriate statement? a. "The drugs are started slowly to minimize side effects." b. "You will be at increased risk for bleeding and infection." c. "High doses will be administered every day for several months." d. "Most patients have more energy and are resistant to infection."

B

A patient with hemophilia is hospitalized with acute knee pain and swelling. What is an appropriate nursing intervention for the patient? a. Wrapping the knee with an elastic bandage b. Placing the patient on bed rest and applying ice to the joint c. Administering nonsteroidal antiinflammatory drugs (NSAIDs) as needed for pain d. Gently performing range-of-motion (ROM) exercises to the knee to prevent adhesions

B

A patient with sickle cell anemia asks the nurse why the sickling crisis does not stop when oxygen therapy is started. Which explanation should the nurse give to the patient? a. Sickling occurs in response to decreased blood viscosity, which is not affected by oxygen therapy. b. When RBCs sickle, they occlude small vessels, which causes more local hypoxia and more sickling. c. The primary problem during a sickle cell crisis is destruction of the abnormal cells, resulting in fewer RBCs to carry oxygen. d. Oxygen therapy does not alter the shape of the abnormal erythrocytes but only allows for increased oxygen concentration in hemoglobin.

B

A patient with stage 2 hypertension who is taking chlorothiazide (Diuril) AND Lisinopril (Zestril) has prazosin (Minipress) added to the medication regimen. what is most important for the nurse to teach the patient to do? a. weigh every morning to monitor for fluid retention b. change position slowly and avoid prolonged standing c. use sugarless gum or candy to help relieve dry mouth d. take the pulse daily to note any slowing of the heart rate

B

A patient with thrombocytopenia with active bleeding is to receive two units of platelets. To administer the platelets, what should the nurse do? a. Check for ABO compatibility. b. Agitate the bag periodically during the transfusion. c. Take vital signs every 15 minutes during the procedure. d. Refrigerate the second unit until the first unit has transfused.

B

In providing care for a patient hospitalized with an acute exacerbation of polycythemia vera, the nurse gives priority to which activity? a. Maintaining protective isolation b. Promoting leg exercises and ambulation c. Protecting the patient from injury or falls d. Promoting hydration with a large oral fluid intake

B

In teaching the patient with pernicious anemia about the disease, the nurse explains that it results from a lack of a. folic acid. b. intrinsic factor. c. extrinsic factor. d. cobalamin intake.

B

Most organ damage in hypertension is related to what? a. Increased fluid pressure exerted against organ tissue b. Atherosclerotic changes in vessels that supply the organs c. Erosion and thinning of blood vessels from constant pressure d. Increased hydrostatic pressure causing leakage of plasma into organ interstitial spaces

B

Nursing interventions for the patient with aplastic anemia are directed toward the prevention of which complications? a. Fatigue and dyspnea b. Hemorrhage and infection c. Thromboemboli and gangrene d. Cardiac dysrhythmias and heart failure

B

The patient who is being admitted has a history of uncontrolled hypertension. High SVR is most likely to cause damage to which organ? a. Brain b. Heart c. Retina d. Kidney

B

The unit is very busy and short staffed. What could the RN delegate to the unlicensed assistive personnel (UAP) ? a. administer antihypertensive medications to stable patients b. obtain orthostatic blood pressure (BP) readings for older patients c. Check BP patients for the patient receiving IV Sodium nitroprusside d. teach about home BP monitoring and use of automatic BP monitoring equipment

B

What causes the anemia of sickle cell disease? a. Intracellular hemolysis of sickled RBCs b. Accelerated breakdown of abnormal RBCs c. Autoimmune antibody destruction of RBCs d. Isoimmune antibody-antigen reactions with RBCs

B

What characteristics should the nurse be aware of in planning care for the patient with Hodgkin's lymphoma? a. Staging of Hodgkin's lymphoma is not important to predict prognosis. b. Nursing management of the patient undergoing treatment for Hodgkin's lymphoma includes measures to prevent infection. c. Hodgkin's lymphoma is characterized by proliferation of malignant activated B cells that destroy the kidneys. d. An important nursing intervention in the care of patients with Hodgkin's lymphoma is increasing fluids to manage hypercalcemia.

B

Which anemia is manifested with pancytopenia? a. Thalassemia b. Aplastic anemia c. Megaloblastic anemia d. Anemia of chronic disease

B

which finding is associated with a blue tinge around the lips and conjunctiva? a. finger clubbing b. central cyanosis c. peripheral cyanosis d. delayed capillary filling time

B

which method is used to evaluate the ECG responses to normal activity over a period of 1 to 2 days a. serial ECGs b. holter monitoring c. 6-min walk test d. event monitor or loop recorder

B

*A nurse is planning to administer a first dose of captopril to a client who has hypertension. Which of the following medications can intensify first dose hypotension?* Select all that apply A. Simvastatin B. Hydrochlorothiazide C. Phenytoin D. Clonidine E. Aliskiren

B , D , E

*When obtaining assessment data from a patient with a microcytic, hypochromic anemia, the nurse would ask the patient about* a. folic acid intake b. dietary intake of iron c. a history of gastric surgery d. a history of sickle cell anemia

B - Iron-deficiency anemia is a microcytic, hypochromic anemia.

*When reviewing the patient's hematologic laboratory values after a splenectomy, the nurse would expect to find* a. RBC abnormalities b. increased WBC count c. decreased hemoglobin d. decreased platelet count

B - Splenectomy can have a dramatic effect in increasing peripheral RBC, white blood cell, and platelet counts.

*A nurse in the emergency department is caring for a client who is having an acute asthma attack. Which of the following assessments indicates that the respiratory status is declining?* Select all that apply A. SaO2 95% B. Wheezing C. Retraction of sternal muscles D. Pink mucous membranes E. Tachycardia

B, C, E

*A nurse is screening a client for hypertension. The nurse should identify that which of the following actions by the client increases his risk for hypertension?* Select all that apply A. Drinking 8 oz nonfat milk daily B. Eating popcorn at the movie theater C. Walking 1 mile daily at 12 min/mile pace D. Consuming 36 oz beer daily E. Getting a massage once a week

B, D

*A nurse is monitoring a client who is receiving epoetin alfa for adverse effects. The nurse should identify which of the following findings as an adverse effect of this medication?* Select all that apply A. Leukocytosis B. Hypertension C. Edema D. Blurred vision E. Headache

B. Hypertension is an adverse effect of epoetin alfa that the nurse should monitor for throughout treatment. E. Headache is an adverse effect of epoetin alfa.

*A nurse is preparing to transfuse a unit of packed red blood cells (PRBCs) for a client who has severe anemia. Which of the following interventions will prevent an acute hemolytic reaction?* A. Ensure that the client has a patent IV line before obtaining blood product from the refrigerator. B. Obtain help from another nurse to confirm the correct client and blood product. C. Take a complete set of vital signs before beginning transfusion and periodically during the transfusion. D. Stay with the client for the first 15 to 30 min of the transfusion.

B. Identifying and matching the correct blood product with the correct client will prevent an acute hemolytic reaction from occurring because this reaction is caused by ABO or Rh incompatibility.

*A nurse is planning to administer subcutaneous enoxaparin 40 mg using a prefilled syringe of enoxaparin 40 mg/0.4 mL to an adult client following hip arthroplasty. Which of the following actions should the nurse plan to take?* A. Expel the air bubble from the prefilled syringe before injecting. B. Insert the needle completely into the client's tissue. C. Administer the injection in the client's thigh. D. Aspirate carefully after inserting the needle into the client's skin.

B. The nurse should inject the needle on the prefilled syringe completely when administering enoxaparin in order to administer the medication by deep subcutaneous injection.

*A nurse is planning to administer IV alteplase to a client. Which of the following interventions should the nurse plan to take?* A. Administer IM enoxaparin along with the alteplase dose. B. Hold direct pressure on puncture sites for up to 30 minutes C. Administer aminocaproic acid IV prior to alteplase infusion. D. Prepare to administer alteplase within 8 hr of manifestation onset.

B. The nurse should plan to hold direct pressure on puncture sites for 10 to 30 min or until oozing of blood stops.

*A nurse is providing instructions to the parent of an adolescent client who has a new prescription for albuterol, PO. Which of the following instructions should the nurse include?* A. "You can take this medication to abort an acute asthma attack." B. "Tremors are an adverse effect of this medication." C. "Prolonged use of this medication can cause hyperglycemia." D. "This medication can slow skeletal growth rate."

B. "Tremors are an adverse effect of this medication."

*A nurse is providing instructions to a client who has a new prescription for albuterol and beclomethasone inhalers for the control of asthma. Which of the following instructions should the nurse include in the teaching?* A. Take the albuterol at the same time each day. B. Administer the albuterol inhaler prior to using the beclomethasone inhaler. C. Use beclomethasone if experiencing an acute episode. D. Avoid shaking the beclomethasone before use.

B. Administer the albuterol inhaler prior to using the beclomethasone inhaler.

*A nurse is assessing a client who is undergoing hemodynamic monitoring. The client has a CVP of 7 mm Hg and a PAWP of 17 mm Hg. Which of the following findings should the nurse expect?* Select all that apply A. Poor skin turgor B. Bilateral crackles in the lungs C. Jugular vein distension D. Dry mucous membranes E. Hepatomegaly

B. Bilateral crackles in the lungs C. Jugular vein distension E. Hepatomegaly

*A nurse is reinforcing teaching with a client on the purpose of taking a bronchodilator. Which of the following client statements indicates an understanding?* A. "This medication can decrease my immune response." B. "I take this medication to prevent asthma attacks." C. "I need to take this medication with food." D. "This medication has a slow onset to treat my symptoms."

B. CORRECT: A bronchodilator can prevent asthma attacks from occurring.

*A nurse is assessing a client who has a history of asthma. Which of the following factors should the nurse identify as a risk for asthma?* A. Gender B. Environmental allergies C. Alcohol use D. History of diabetes

B. CORRECT: Environmental allergies are a risk factor associated with asthma. A client who has environmental allergies typically has other allergic problems, such as rhinitis or a skin rash

*A nurse is monitoring a client who takes aspirin daily. The nurse should identify which of the following manifestations as adverse effects of aspirin?* Select all that apply. A. Hypertension B. Coffee-ground emesis C. Tinnitus D. Paresthesias of the extremities E. Nausea

B. CORRECT: GI bleeding with dark stools or coffee-ground emesis can be an adverse effect of aspirin therapy. C. CORRECT: Tinnitus and hearing loss can occur as an adverse effect of aspirin therap E. CORRECT: Nausea, vomiting, and abdominal pain can occur as a result of aspirin therapy.

*A nurse is planning care for for a client who has a new prescription for torsemide. The nurse should plan to monitor which of the following conditions as potential adverse reactions of this medications?* Select all that apply A. Respiratory acidosis B. Hypokalemia C. Hypotension D. Ototoxicity E. Ventricular dysrhythmias

B. Hypokalemia C. Hypotension D. Ototoxicity E. Ventricular dysrhythmias

*A nurse is caring for a client who is admitted to the emergency department with a blood pressure of 266/147 mmHg. The client reports a headache and double vision. The client states, "I ran out of my diltiazem 3 days ago, and I am unable to purchase more." Which of the following actions should the nurse take first?* A. Administer acetaminophen for headache B. Provide teaching regarding the importance of not abruptly stopping an antihypertensive C. Obtain IV access and prepare to administer an IV antihypertensive D. Call social services for a referral for financial assistance in obtaining prescribed medication

C

*The major advantage of a Venturi mask is that it can?* A. Deliver up to 80% O2 B. Provide continuous 100% humidity C. Deliver a precise concentration of O2 D. Be used while a patient eats and sleeps

C

A 38-yr-old man is treated for hypertension with triamterene and hydrochlorothiazide and metropolol (lopressor). four months after his last clinic visit, his BP returns to pretreatment levels and he admits he has not been taking his medication regularly . What is the nurse's best response to his patient? a. "Try always to take your medication when you carry out another daily routine so that you do not forget to take it" b. "You probably would not need to take medications for hypertension if you would exercise more and stop smoking" c. "The drugs you are taking cause sexual dysfunction in many patients. Are you experiencing any problems in this area?" d. "You need to remember that hypertension can be only controlled with medicaiton, not cured, and you must always take your medication"

C

A patient is admitted to the hospital for evaluation and treatment of thrombocytopenia. Which action is most important for the nurse to implement? a. Taking the temperature every 4 hours to assess for fever b. Maintaining the patient on strict bed rest to prevent injury c. Monitoring the patient for headaches, vertigo, or confusion d. Removing the oral crusting and scabs with a soft brush four times a day

C

A patient with acute myelogenous leukemia is considering a hematopoietic stem cell transplant and asks the nurse what is involved. What is the best response the nurse can give the patient? a. "Your bone marrow is destroyed by radiation and new bone marrow cells from a matched donor are injected into your bones." b. "A specimen of your bone marrow may be aspirated and treated to destroy any leukemic cells and then reinfused when your disease becomes worse." c. "Leukemic cells and bone marrow stem cells are eliminated with chemotherapy and/or total-body radiation and new bone marrow cells from a donor are infused." d. "During chemotherapy and/or total-body irradiation to destroy all of your blood cells, you may be given transfusions of red blood cells and platelets to prevent complications."

C

A patient with hemophilia comes to the clinic for treatment. What should the nurse anticipate that he or she will need to administer? a. Whole blood b. Thromboplastin c. Coagulation factor d. Fresh frozen plasma

C

During the assessment of a patient with cobalamin deficiency, what manifestation would the nurse expect to find in the patient? a. Icteric sclera b. Hepatomegaly c. Paresthesia of the hands and feet d. Intermittent heartburn with acid reflux

C

Following a splenectomy for the treatment of ITP, the nurse would expect the patient's laboratory test results to reveal which of the following? a. Decreased RBCs b. Decreased WBCs c. Increased platelets d. Increased immunoglobulins

C

The patient diagnosed with secondary hypertension asks why it is called secondary and not primary. What is the best explanation for the nurse to provide? A. Has a more gradual onset than primary hypertension. B. Does not cause the target organ damage that occurs primary hypertension. C. Has a specific cause , such as renal disease, that often can be treated by medicine or surgery. D. Is caused by age-related changes in BP regulatory mechanisms in people over 65 years of age

C

What is the primary BP effect of B-adrenergic blockers such as atenolol ( Tenormin)? a. Vasodilation of arterioles by blocking movement of calcium into cells b. Decrease Na+ and water reabsorption by blocking the effect of aldosterone c. Decrease CO by decreasing rate and strength of the heart and renin secretion by the kidneys d. Vasodilation caused by inhibiting sympathetic outflow from the central nervous system (CNS)

C

What is the underlying cause of lymphadenopathy, splenomegaly, and hepatomegaly in leukemia? a. The development of infection at these sites b. Increased compensatory production of blood cells by these organs c. Infiltration of the organs by increased numbers of WBCs in the blood d. Normal hypertrophy of the organs in an attempt to destroy abnormal cells

C

Which manifestation is an indication that a patient is having hypertensive emergency? a. symptoms of a stroke with an elevated BP b. a systolic BP>180mm Hg and a diastolic BP>110mm Hg c. a sudden rise in BP accompanied by neurologic impairment d. a severe elevation of BP that occurs over several days or weeks

C

Which type of transfusion reaction occurs with leukocyte or plasma protein incompatibility and may be avoided with leukocyte reduction filters? a. Allergic reaction b. Acute hemolytic reaction c. Febrile, nonhemolytic reaction d. Massive blood transfusion reaction

C

the nursing student is seeking assistance in hearing the patient's abnormal heart sounds. what should the nurse tell the student to do for a more effective assessment? a. use the diaphragm of the stethoscope with the patient prone. b. use the diaphragm of the stethoscope with the patient supine c. use the bell of the stethoscope with the patient leaning forward d. use the bell of the stethoscope with the patient on the right side

C

when caring for a patient after a cardiac catheterization with coronary angiography, which finding should be of most concern to the nurse? a. swelling at the catheter insertion site b. development of raised wheals on the patient's trunk c. absence of pulses distal to the catheter insertion site d. patient pain at the insertion site at 4 on a scale of 0-10

C

*A nurse is caring for a hospitalized client who has an activated partial thromboplastin time (aPTT) greater than 1.5 times the expected reference range. Which of the following blood products should the nurse prepare to transfuse?* A. Whole blood B. Platelets C. Fresh frozen plasma D. Packed red blood cells

C Fresh frozen plasma is indicated for a client who has an elevated aPTT because it replaces coagulation factors and can help prevent bleeding.

*A nurse in an urgent care clinic is obtaining a history from a client who has type 2 diabetes mellitus and a recent diagnosis of hypertension. This is the second time in 2 weeks that the client experienced hypoglycemia. Which of the following client data should the nurse report to the provider?* A. Takes psyllium daily as a fiber laxative B. Drinks skim milk daily as a bedtime snack C. Takes metoprolol daily after meals D. Drinks grapefruit juice daily with breakfast

C metoprolol is a beta blocker that can mask the effects of hypoglycemia in clients who have diabetes mellitus

*A nurse in an acute care facility is caring for a client who is receiving IV nitroprusside for hypertensive crisis. Which of the following conditions should the nurse monitor the client for as an adverse effect of this medication?* A. Intestinal ileus B. Neutropenia C. Delirium D. Hyperthermia

C can alter mental status changes can occur with this drug when high does are given via IV

*A nurse is caring for a client who a new prescription for captopril for hypertension. The nurse should monitor the client for which of the following as an adverse effect of this medication?* A. Hypokalemia B. Hypernatremia C. Neutropenia D. Bradycardia

C is a serious adverse effect that can occur in clinets taking an ACE inhibitor.

*A patient with multiple myeloma becomes confused and lethargic. The nurse would expect that these clinical manifestations may be explained by diagnostic results that indicate* a. hyperkalemia b. hyperuricemia c. hypercalcemia d. CNS myeloma

C - Bone degeneration in multiple myeloma causes calcium to be lost from bones, which eventually results in hypercalcemia. Hypercalcemia may cause renal, gastrointestinal, or neurologic manifestations, such as polyuria, anorexia, or confusion, and may ultimately cause seizures, coma, and cardiac problems.

*DIC is a disorder in which* a. the coagulation pathway is genetically altered, leading to thrombus formation in all major blood vessels b. an underlying disease depletes hemolytic factors in the blood, leading to diffuse thrombotic episodes and infarcts c. a disease process stimulates coagulation processes with resultant thrombosis, as well as depletion of clotting factors, leading to diffuse clotting and hemorrhage d. an inherited predisposition causes a deficiency of clotting factors that leads to overstimulation of coagulation processes in the vasculature

C - In disseminated intravascular coagulation (DIC), the coagulation process is stimulated, with resultant thrombosis and depletion of clotting factors, which leads to diffuse clotting and hemorrhage. The paradox of this condition is characterized by the profuse bleeding that results from the depletion of platelets and clotting factors.

*The nurse is aware that a major difference between Hodgkin's lymphoma and non-Hodgkin's lymphoma is that* a. Hodgkin's lymphoma occurs only in young adults b. Hodgkin's lymphoma is considered potentially curable c. non-Hodgkin's lymphoma can manifest in multiple areas d. non-Hodgkin's lymphoma is treated only with radiation therapy

C - Non-Hodgkin's lymphoma can originate outside the lymph nodes, the method of spread can be unpredictable, and most affected patients have widely disseminated disease.

*In a severely anemic patient, the nurse would expect to find:* a. cyanosis and pulmonary edema b. pulmonary edema and fibrosis c. dyspnea at rest and tachycardia d. ventricular dysrhythmias and wheezing

C - Patients with severe anemia (hemoglobin level, less than 6 g/dL) exhibit the following cardiovascular and pulmonary manifestations: tachycardia, increased pulse pressure, systolic murmurs, intermittent claudication, angina, heart failure, myocardial infarction, tachypnea, orthopnea, and dyspnea at rest.

*Which medications would be most appropriate to administer to a patient experiencing an acute asthma attack?* Select all that apply A. montelukast (Singulair) B. inhaled hypertonic saline C. albuterol (Proventil HFA) D. ipratropium (Atrovent HFA) E. salmeterol (Serevent Diskus)

C, D

*A nurse is assessing a client during transfusion of a unit of whole blood. The client develops a cough, shortness of breath, elevated blood pressure, and distended neck veins. The nurse should anticipate a prescription for which of the following medications?* A. Epinephrine B. Lorazepam C. Furosemide D. Diphenhydramine

C. Furosemide, a loop diuretic, may be prescribed to relieve manifestations of circulatory overload.

*A nurse is caring for a client who has atrial fibrillation and a new prescription for dabigatran. Which of the following medications is prescribed concurrently to treat an adverse effect of dabigatran?* A. Vitamin K1 B. Protamine C. Omeprazole D. Probenecid

C. Omeprazole or another proton pump inhibitor is prescribed for a client who is taking dabigatran and has abdominal pain and other GI findings that can occur as adverse effects of dabigatran. The nurse should advise the client who has GI effects to take dabigatran with food.

*A nurse is caring for a hospitalized client who is receiving IV heparin for a deep-vein thrombosis. The client begins vomiting blood. After the heparin has been stopped, which of the following medications should the nurse prepare to administer?* A. Vitamin K1 B. Atropine C. Protamine D. Calcium gluconate

C. Protamine reverses the anticoagulant effect of heparin.

*A nurse is caring for a client who is receiving daily doses of oprelvekin. Which of the following laboratory values should the nurse monitor to determine effectiveness of this medication?* A. Hemoglobin B. Absolute neutrophil count C. Platelet count D. Total white blood count

C. The expected outcome for oprelvekin is a platelet count greater than 50,000/mm3.

*A nurse is providing discharge teaching to a client who has COPD and a new prescription for albuterol. Which of the following statements by the client indicates an understanding of the teaching?* A. "This medication can increase my blood sugar levels." B. "This medication can decrease my immune response." C. "I can have an increase in my heart rate while taking this medication." D. "I can have mouth sores while taking this medication."

C. CORRECT: Bronchodilators, such as albuterol, can cause tachycardia.

*A nurse is planning to instruct a client on how to perform pursed‑lip breathing. Which of the following should the nurse include in the plan of care?* A. "Take quick breaths upon inhalation." B. "Place your hand over your stomach." C. "Take a deep breath in through your nose." D. "Puff your checks upon exhalation."

C. CORRECT: The client should take a deep breath in through her nose while performing pursed‑lip breathing. This controls the client's breathing.

*A nurse is providing discharge teaching to a client who has a new prescription for prednisone for asthma. Which of the following client statements indicates an understanding of the teaching?* A. "I will decrease my fluid intake while taking this medication." B. "I will expect to have black, tarry stools." C. "I will take my medication with meals." D. "I will monitor for weight loss while on this medication."

C. CORRECT: The client should take this medication with food. Taking prednisone on an empty stomach can cause gastrointestinal distress.

*A nurse is caring for a client who has increased intracranial pressure and is receiving mannitol. Which the following findings should the nurse report to the provider?* A. Blood glucose 150 mg/dL B. Urine output 40 mL/hr C. Dyspnea D. Bilateral equal pupil size

C. Dyspnea

*A nurse is monitoring a client who is receiving spironolactone. Which of the following findings should the nurse report to the provider?* A. Blood sodium 144 mEq/L B. Urine output 120 mL in 4 hr C. Blood potassium 5.2 mEq/l D. Blood pressure 140/90 mm Hg

C. Serum potassium 5.2 mEq/l

*A priority consideration in the management of the older adult with hypertension is to* a. prevent primary hypertension from converting to secondary hypertension. b. recognize that the older adult is less likely to adhere to the drug therapy regimen than a younger adult. c. ensure that the patient receives larger initial doses of antihypertensive drugs because of impaired absorption. d. use precise technique in assessing the BP of the patient because of the possible presence of an auscultatory gap.

D

*In teaching a patient with hypertension about controlling the illness, the nurse recognizes that* a. all patients with elevated BP require medication. b. obese persons must achieve a normal weight to lower BP. c. it is not necessary to limit salt in the diet if taking a diuretic. d. lifestyle modifications are indicated for all persons with elevated BP.

D

*Which of the following guidelines would be a part of teaching patients how to use a metered-dose inhaler (MDI)?* A. After activating the MDI, breathe in as quickly as you can B. Estimate the remaining amount of medicine in the MDI by floating the canister in water C.Disassemble the plastic canister from the inhaler and rinse both pieces under running water every week D. To determine how long the canister will last, divide the total number of puffs in the canister by the puffs needed per day

D

A 20-year-old female patient is in the emergency department for anorexia and fatigue. She takes phenytoin (Dilantin) for a seizure disorder and oral contraceptives. Which type of anemia is this patient most at risk for? a. Aplastic anemia b. Hemolytic anemia c. Iron-deficiency anemia d. Folic acid deficiency anemia

D

At an outpatient clinic, K.L.'s 78-year-old grandma is found to have a Hgb of 8.7 g/dL (87 g/L) and a Hct of 35%. Based on the most common cause of these findings in the older adult, the nurse collects information in the older adult regarding a. a history of jaundice and black tarry stools. b. a 3-day diet recall of the foods the patient has eaten. c. any drugs that have depressed the function of the bone marrow. d. a history of any chronic diseases such as cancer or renal disease.

D

Dietary teaching that includes eating eating dietary sources of potassium is indicated for the hypertensive patient taking which drug? a. Enalapril b. Labetalol c. Spironolactone d. Hydrochlorothiazide

D

During discharge teaching of a patient with newly diagnosed sickle cell disease, what should the nurse teach the patient to do? a. Limit fluid intake b. Avoid humid weather c. Eliminate exercise from the lifestyle d. Seek early medical intervention for upper respiratory infections

D

During the physical assessment of the patient with severe anemia, which finding is of the most concern to the nurse? a. Anorexia b. Bone pain c. Hepatomegaly d. Dyspnea at rest

D

In reviewing the laboratory results of a patient with hemophilia A, what would the nurse expect to find? a. An absence of factor IX b. A decreased platelet count c. A prolonged bleeding time d. A prolonged partial thromboplastin time (PTT)

D

The patient asks the nurse about valsartan (Diovan), The new medication prescribed for blood pressure. What is the best explanation the nurse can use to explain the action of this medication? a. prevents the conversion of angiotensin I to angiotensin II b. acts directly on smooth muscle of arterioles to cause vasodilation c. decreases extracellular fluid volume by increasing Na+ and Cl- excretion with water d. vasodilation, prevents the action of angiotensin II, and promotes increased salt and water excretion

D

The strict vegetarian is at highest risk for the development of which anemia? a. Thalassemia b. Iron-deficiency anemia c. Folic acid deficiency anemia d. Cobalamin deficiency anemia

D

What does the nursing responsibility in the management of the patient with hypertensive urgency often include? a. Monitoring hourly urine output for drug effectiveness b. Titrating IV drug dosages based on BP measurements every 2 to 3 minutes c. Providing continuous electrocardiographic (ECG) monitoring to detect side effects of the drugs d. Instructing the patient to follow up with a health care professional within 24 hours after outpatient treatment

D

What is a major method of preventing infection in the patient with neutropenia? a. Prophylactic antibiotics b. A diet that eliminates fresh fruits and vegetables c. High-efficiency particulate air (HEPA) filtration rooms d. Strict hand washing by all persons in contact with the patient

D

What is a nursing intervention that is indicated for the patient during a sickle cell crisis? a. Frequent ambulation b. Application of antiembolism hose c. Restriction of sodium and oral fluids d. Administration of large doses of continuous opioid analgesics

D

What is included in the correct technique for BP measurements? a. always take the BP in both arms b. position the patient supine for all readings c. place the cuff loosely around the upper arm d. take readings at least two times at least 1 minute apart

D

Which bleeding disorder affects both genders, is autosomal dominant, and will have laboratory results showing prolonged bleeding time? a. Hemophilia A b. Hemophilia B c. Thrombocytopenia d. von Willebrand disease

D

Which drugs are most commonly used to treat hypertensive crises? a. Labetalol and bumetanide (Bumex) b. Esmolol (Brevibloc) and captopril (Captopril) c. Enalaprilat (Vasotec) and minoxidil (Minoxidil) d. Fenoldopam (Corlopam) and sodium nitroprusside (Nitropress)

D

Which leukemia is seen in 80% of adults with acute leukemia and exhibits proliferation of precursors of granulocytes? a. Hairy cell leukemia b. Biphenotypic leukemia c. Acute lymphocytic leukemia (ALL) d. Acute myelogenous leukemia (AML)

D

a female patient has a total cholesterol level of 232 mg/dl (6.0 mmol/L) and a high-density lipoprotein (HDL) of 65 mg/dl (1.68mmol/L). A male patient has a total cholesterol level of 200mg/dl (5.172mmol/L) and an HDL of 32mg/dl (0.83mmol/L). based on these findings, which patient has the highest cardiac risk? a. the man, because his HDL is lower b. the woman, because her HDL is higher c. the woman, because her cholesterol is higher d. the man, because his cholesterol-to-HDL ratio is higher

D

the nurse caring for a patient immediately following a transesophageal echocardiogram (TEE) should consider which action the highest priority? a. monitor the ECG b. monitor pulse oximetry c. assess vital signs (BP, HR, RR, Temp) d. maintain NPO status until gag reflex has returned

D

when palpating the patient's popliteal pulse, the nurse feels a vibration at the site. How should the nurse record this finding? a. thready, weak pulse b. bruit at the artery site c. bounding pulse volume d. thrill of the popliteal artery

D

*The most common type of leukemia in adults in western countries is* a. acute myelocytic leukemia b. acute lymphocytic leukemia c. chronic myelocytic leukemia d. chronic lymphocytic leukemia

D - Chronic lymphocytic leukemia is a disease primarily of older adults.

*Multiple drugs are often used in combinations to treat leukemia and lymphoma because* a. there are fewer toxic side effects b. the chance that one drug will be effective is increased c. the drugs are more effective without causing side effects d. the drugs work by different mechanisms to maximize killing of cancer cells

D - Combination therapy is the mainstay of treatment for leukemia. The three purposes for using multiple drugs are to (1) decrease drug resistance, (2) minimize the drug toxicity to the patient by using multiple drugs with varying toxic effects, and (3) interrupt cell growth at multiple points in the cell cycle.

*Complications of transfusions that can be decreased by the use of leukocyte depletion or reduction of RBC transfusion are* a. chills and hemolysis b. leukostasis and neutrophilia c. fluid overload and pulmonary edema d. transmission of cytomegalovirus and fever

D - Infectious viruses, such as human immunodeficiency virus (HIV), human herpesvirus, hepatitis B and C type 6 (HCV-6), Epstein-Barr virus (EBV), human T-cell leukemia virus type 1 (HTLV-1), and cytomegalovirus (CMV), and other agents, such as the agent that causes malaria, can be transmitted by blood transfusion. Leukocyte-reduced blood products drastically reduce the risk for viral infections associated with blood transfusions, including CMV.

*Because myelodysplastic syndrome arises from the pluripotent hematopoietic stem cell in the bone marrow, laboratory results the nurse would expect to find include a(n)* a. excess of T cells b. excess of platelets c. deficiency of granulocytes d. deficiency of all cellular blood components

D - Myelodysplastic syndrome (MDS) commonly manifests as infection and bleeding. It is caused by inadequate numbers of ineffective functioning circulating granulocytes or platelets.

*The nurse would expect that a patient with von Willebrand disease undergoing surgery would be treated with administration of vWF and* a. thrombin b. factor VI c. factor VII d. factor VIII

D - von Willebrand disease involves deficiency of the von Willebrand coagulation protein, variable factor VIII deficiencies, and platelet dysfunction. Treatment includes administration of von Willebrand factor and factor VIII.

*A nurse is preparing to administer filgrastim for the first time to a client who has just undergone a bone marrow transplant. Which of the following actions should the nurse take?* A. Administer IM in a large muscle mass to prevent injury. B. Ensure that the medication is refrigerated until just prior to administration. C. Shake vial gently to mix well before withdrawing dose. D. Discard vial after removing one dose of the medication.

D. Only one dose of filgrastim should be withdrawn from the vial and the vial should then be discarded.

*A nurse is orienting a newly licensed nurse on the care of a client who is to have a line placed for hemodynamic monitoring. Which of the following statements by the newly licensed nurse indicates understanding?* A. "Air should be instilled into the monitoring system prior to the procedure." B. "The client should be positioned on the left side during the procedure." C. "The transducer should be level with the second intercostal space after the line is placed." D. "A chest x‑ray is needed to verify placement after the procedure."

D. "A chest x‑ray is needed to verify placement after the procedure."

*A nurse is teaching a client who is scheduled for a coronary angiography. Which of the following statements should the nurse include?* A. "You should have nothing to eat or drink for 4 hours prior to the procedure." B. "You will be given general anesthesia during the procedure." C. "You should not have this procedure done if you are allergic to eggs." D. "You will need to keep your affected leg straight following the procedure."

D. "You will need to keep your affected leg straight following the procedure."

*A nurse is instructing a client on the use of an incentive spirometer. Which of the following statements by the client indicates an understanding of the teaching?* A. "I will place the adapter on my finger to read my blood oxygen saturation level." B. "I will lie on my back with my knees bent." C. "I will rest my hand over my abdomen to create resistance." D. "I will take in a deep breath and hold it before exhaling."

D. CORRECT: The client who is using the spirometer should take in a deep breath and hold it for 3 to 5 seconds

*A nurse is caring for a client 2 hr after admission. The client has an SaO2 of 91%, exhibits audible wheezes, and is using accessory muscles when breathing. Which of the following classes of medications should the nurse expect to administer?* A. Antibiotic B. Beta‑blocker C. Antiviral D. Beta2 agonist

D. CORRECT: The nurse should administer a beta2 agonist, which causes dilation of the bronchioles to relieve symptoms

A patient arrives at an urgent care center after experiencing unrelenting substernal and epigastric pain and pressure for about 12 hours. The nurse reviews laboratory results with the understanding that at this point in time, a myocardial infarction would by indicated by peak levels of a. Troponin T. b. Homocysteine. c. Creatine kinase-MB. d. Type b natriuretic peptide.

a

A patient is admitted to the emergency department with a severe exacerbation of asthma. Which finding is of most concern to the nurse? a. Unable to speak and sweating profusely b. PaO2 of 80 mm Hg and PaCO2 of 50 mm Hg c. Presence of inspiratory and expiratory wheezing d. Peak expiratory flow rate at 60% of personal best

a

A patient is undergoing a contrast computed tomography (CT) of the spleen. What is most important for the nurse to ask the patient before the test? a. Iodine sensitivity b. Prior blood transfusions c. Phobia of confined spaces d. Internal metal implants or appliances

a

After a woman had a right breast mastectomy, her right arm became severely swollen. What hematologic problem caused this? a. Lymphedema b. Right-sided heart failure c. Wound on her right hand d. Refusal to use her right arm

a

During the nursing assessment of a patient with anemia, what specific information should the nurse ask the patient about? a. Stomach surgery b. Recurring infections c. Corticosteroid therapy d. Oral contraceptive use

a

The nurse is caring for a patient with COPD. Which intervention could be delegated to unlicensed assistive personnel (UAP)? a.Assist the patient to get out of bed. b.Auscultate breath sounds every 4 hours. c.Plan patient activities to minimize exertion. d.Teach the patient pursed lip breathing technique

a

The nurse takes blood pressures at a health fair. The nurse identifies which person as most at risk for developing hypertension? a. A 52-year-old male who smokes and has a parent with hypertension b. A 30-year-old female advertising agent who is unmarried and lives alone c. A 68-year-old male who uses herbal remedies to treat an enlarged prostate gland d. A 43-year-old female who travels extensively for work and exercises only on weekends

a

Using light pressure with the index and middle fingers, the nurse cannot palpate any of the patient's superficial lymph nodes. How should the nurse respond to this assessment? a. Record this finding as normal b. Reassess the lymph nodes using deeper pressure c. Ask the patient about any history of radiation therapy d. Notify the health care provider that x-rays of the nodes will be necessary.

a

When teaching the patient about a new prescription for oral iron supplements, what does the nurse instruct the patient to do? a. Increase fluid and dietary fiber intake b. Take the iron preparations with meals c. Use enteric-coated preparations taken with orange juice d. Report the presence of black stools to the health care provider

a

Which method of low, constant oxygen administration is the safest system to use for a patient with COPD exacerbation? a.Venturi mask b.Nasal cannula c.Simple face mask d.Non-rebreathing mask

a

What are the characteristics of neutrophils (select all that apply)? a. Also known as "segs" b. Band is immature cell c. First WBC at injury site d. Arises from megakaryocyte e. Increased in individuals with allergies f. 60% to 70% of WBCs

a b c f

A patient with a bone marrow disorder has an overproduction of myeloblasts. The nurse would expect the results of a complete blood count (CBC) to include an increase in which cell types (select all that apply)? a. Basophils b. Eosinophils c. Monocytes d. Neutrophils e. Lymphocytes

a b d

A patient who was in a car accident had abdominal trauma. Which organs may be damaged and contribute to altered function of the hematologic system (select all that apply)? a. Liver b. Spleen c. Stomach d. Gallbladder e. Lymph nodes

a b e

Which nutrients are essential for red blood cell production (select all that apply)? a. Iron b. Folic acid c. Vitamin C d. Vitamin D e. Vitamin B12 f. Carbohydrates

a b e

increases in which blood studies are diagnostic for acute coronary syndrome (ACS) (select all that apply) a. copeptin b. creatine kinase (CK-MM) c. cardiac troponin T (cTnT) d. b-type natriuretic peptide (BNP) e. high sensitivity C-reactive protein (hs-CRP) f. lipoprotein-associated phospholipase A2 (Lp-PLA2)

a c

Which cells are classified as granulocytes (select all that apply)? a. Basophil b. Monocyte c. Eosinophil d. Neutrophil e. Lymphocyte f. thrombocyte

a c d

Which characteristics are related to an acute hemolytic transfusion reaction (select all that apply)? a. ABO incompatibility b. Hypothermia common c. Destruction of donor RBCs d. Acute kidney injury occurs e. Hypocalcemia and hyperkalemia f. Epinephrine used for severe reaction

a c d

Which statements accurately describe thrombocytopenia (select all that apply)? a. Patients with platelet deficiencies can have internal or external hemorrhage. b. The most common acquired thrombocytopenia is thrombotic thrombocytopenic purpura (TTP). c. Immune thrombocytopenic purpura (ITP) is characterized by increased platelet destruction by the spleen. d. TTP is characterized by decreased platelets, decreased RBCs, and enhanced aggregation of platelets. e. A classic clinical manifestation of thrombocytopenia that the nurse would expect to find on physical examination of the patient is ecchymosis.

a c d

What are the nonmodifiable risk factors for primary hypertension? A- Age B- Obesity C- Gender D- Ethnicity E- Genetic link

a c d e

14. A patient with aplastic anemia has a nursing diagnosis of impaired oral mucous membrane. The etiology of this diagnosis can be related to the effects of what deficiencies (select all that apply)? a. RBCs b. Ferritin c. Platelets d. Coagulation factor VIII e. White blood cells (WBCs)

a c e

A 78-yr-old patient is admitted with a BP of 180/98 mm Hg. which age related physical changes may contribute to this patient's hypertension (select all that apply)? a. decreased renal function b. increased baroreceptor reflexes c. increased peripheral vascular resistance d. increased adrenergic receptor sensitivity e. increased collagen and stiffness of the myocardium f. loss of elasticity in large arteries from arteriosclerosis

a c e f

Which statements accurately describe chronic lymphocytic leukemia (select all that apply)? a. Most common leukemia of adults b. Only cure is bone marrow transplant c. Neoplasm of activated B lymphocytes d. Increased incidence in survivors of atomic bombs e. Philadelphia chromosome is a diagnostic hallmark f. Mature-appearing but functionally inactive lymphocytes

a c f

Which statements describe anemia related to blood loss (select all that apply)? a. A major concern is prevention of shock. b. This anemia is most frequently treated with increased dietary iron intake. c. In addition to the general symptoms of anemia, this patient also manifests jaundice. d. Initial clinical symptoms are the most reliable way to evaluate the effect and degree of blood loss. e. A patient who has acute blood loss may have postural hypotension and increased heart rate.

a d e

Which statements by a patient with moderate asthma inform the nurse that the patient needs more teaching about medications (select all that apply)? a."If I can't afford all of my medicines, I will only use the salmeterol (Serevent)." b."I will stay inside if there is a high pollen count to pre vent having an asthma attack." c."I will rinse my mouth after using fluticasone (Flo vent HFA) to prevent oral candidiasis." d."I must have omalizumab (Xolair) injected every 2 to 4 weeks because inhalers don't help my asthma." e."I can use my inhaler three times, every 20 minutes, before going to the hospital if my peak flow has not improved." f."My gastroesophageal reflux disease (GERD) medications will help my asthma and my asthma medications will help my GERD."

a f

*A nurse is providing discharge teaching to a client who had a gastrectomy for stomach cancer. Which of the following information should the nurse include in the teaching?* Select all that apply a. "You will need a monthly injection of vitamin B12 for the rest of your life." b. "Using the nasal spray form of vitamin B12 taken on a daily basis is an option." c. "An oral supplement of vitamin B12 taken on a daily basis can be an option." d. "You should increase your intake of animal proteins, legumes and dairy products to increase vitamin B12 in your diet." e. "Add soy milk fortified with vitamin B12 to your diet to decrease the risk of pernicious anemia."

a. "You will need a monthly injection of vitamin B12 for the rest of your life." b. "Using the nasal spray form of vitamin B12 taken on a daily basis is an option."

Number the components of normal hemostasis in the order of occurrence, beginning with 1 for the first component and ending with 4 for the last component. _________a. Lysis of clot _________b. Vascular response _________c. Plasma clotting factors _________d. Platelet plug formation

a. 4 b. 1 c. 3 d. 2

*A patient has a severe blockage in his right coronary artery. Which heart structures are most likely to be affected by this blockage?* Select all that apply a. AV node b. Left ventricle c. Coronary sinus d. Right ventricle e. Pulmonic valve

a. AV node b. Left ventricle d. Right ventricle Rationale: The right coronary artery (RCA) supplies blood to the right atrium, the right ventricle, and a portion of the posterior wall of the left ventricle. In 90% of people, the RCA supplies blood to the atrioventricular (AV) node, bundle of His, and part of the cardiac conduction system.

*A nurse is planning care for a client who is receiving furosemide IV for peripheral edema. Which of the following interventions should the nurse include in the plan of care?* Select all that apply a. Assess for tinnitus b. Report urine output 50 mL/hr. c. Monitor blood potassium levels. d. Elevate the head of the bed slowly before ambulation e. Recommend eating a banana daily

a. Assess for tinnitus c. Monitor blood potassium levels. d. Elevate the head of the bed slowly before ambulation e. Recommend eating a banana daily

*A nurse is caring for a client who has DIC. Which of the following medications should the nurse anticipate administering?* a. Heparin b. Vitamin K c. Mefoxin d. Simvastatin

a. Heparin

*Which nursing responsibilities are priorities when caring for a patient returning from a cardiac catheterization* Select all that apply a. Monitoring vital signs and ECG b. Checking the catheter insertion site and distal pulses c. Assisting the patient to ambulate to the bathroom to void d. Informing the patient that he will be sleepy from the general anesthesia e. Instructing the patient about the risks of the radioactive isotope injection

a. Monitoring vital signs and ECG b. Checking the catheter insertion site and distal pulses Rationale: The nursing responsibilities after cardiac catheterization include assessment of the puncture site for hematoma and bleeding; assessment of circulation to the extremity used for catheter insertion and of peripheral pulses, color, and sensation of the extremity; and monitoring vital signs and electrocardiographic rhythm. Other nursing responsibilities are described in Table 31-6.

*A nurse is caring for a client who has disseminated intravascular coagulation (DIC). Which of the following laboratory values indicate the client's clotting factors are depleted?* Select all that apply a. Platelets 100,000 mm3 b. Fibrinogen levels 120 mg/dL c. Fibrin degradation products 4.3 mcg/mL d. D-dimer 0.03 mcg/mL e. Sedimentation rate 38 mm/hr

a. Platelets 100,000 mm3 b. Fibrinogen levels 120 mg/dL

*A nurse is planning care for a client who has Hgb 7.5 g/dL and HCT 21.5%. Which of the following actions should the nurse include in the plan of care?* Select all that apply a. Provide assistance with ambulation b. Monitor oxygen saturation c. Weigh the client weekly d. Obtain stool specimen for a occult blood e. Schedule daily rest periods

a. Provide assistance with ambulation b. Monitor oxygen saturation d. Obtain stool specimen for a occult blood e. Schedule daily rest periods

*A nurse is caring for a client who is receiving a blood transfusion. Which of the following actions should the nurse plan to take if an allergic transfusion reaction is suspected?* Select all that apply a. Stop the transfusion b. Monitor for hypertension c. Maintain an IV infusion with 0.9% sodium chloride d. Position client in an upright position with the feet lower than the heart e. Administer diphenhydramine

a. Stop the transfusion c. Maintain an IV infusion with 0.9% sodium chloride e. Administer diphenhydramine

*When a person's blood pressure rises, the homeostatic mechanism to compensate for an elevation involves stimulation of* a. baroreceptors that inhibit the sympathetic nervous system, causing vasodilation b. chemoreceptors that inhibit the sympathetic nervous system, causing vasodilation c. baroreceptors that inhibit the parasympathetic nervous system, causing vasodilation d. chemoreceptors that stimulate the sympathetic nervous system, causing an increased heart rate

a. baroreceptors that inhibit the sympathetic nervous system, causing vasodilation Rationale: Baroreceptors in the aortic arch and carotid sinus are sensitive to stretch or pressure within the arterial system. Stimulation of these receptors sends information to the vasomotor center in the brainstem. This results in temporary inhibition of the sympathetic nervous system and enhancement of the parasympathetic influence, which cause a decrease in heart rate and peripheral vasodilation.

*When assessing a patient, you note a pulse deficit of 23 beats. This finding may be caused by* a. dysrhythmias b. heart murmurs c. gallop rhythms d. pericardial friction rubs

a. dysrhythmias Rationale: A pulse deficit occurs if there is a difference between the apical and radial beats per minute. A pulse deficit indicates cardiac dysrhythmias.

*Nursing care for a patient immediately after a bone marrow biopsy and aspiration includes* Select all that apply a. giving analgesics as needed b. preparing to start a blood transfusion c. giving preprocedure and postprocedure antibiotic medications d. having the patient lie still to keep the sterile pressure dressing intact e. monitoring vital signs and assessing the site for excess drainage or bleeding

a. giving analgesics as needed d. having the patient lie still to keep the sterile pressure dressing intact e. monitoring vital signs and assessing the site for excess drainage or bleeding

*Significant information from the patient's health history that relates to the hematologic system includes* a. jaundice b. bladder surgery c. early menopause d. multiple pregnancies

a. jaundice

*When reviewing laboratory results of an older patient with an infection, the nurse would expect to find* a. mild leukocytosis b. decreased platelet count c. increased hemoglobin and hematocrit levels d. decreased erythrocyte sedimentation rate (ESR)

a. mild leukocytosis

Lab test results indicate increased fibrin split products (FSPs). An appropriate nursing action is to monitor the patient for a. fever. b. bleeding. c. faintness. d. thrombotic episodes.

b

A patient is admitted to the emergency department with an acute asthma attack. Which patient assessment is of greatest concern to the nurse? a.The presence of a pulsus paradoxus b.Markedly diminished breath sounds with no wheezing c.Use of accessory muscles of respiration and a feeling of suffocation d.A respiratory rate of 34 and increased pulse and blood pressure

b

A patient with a hematologic disorder has a smooth, shiny, red tongue. Which lab result would the nurse expect to see? a. Neutrophils 45% b. Hgb 9.6 g/dL (96 g/L) c. WBC count 13,500/uL d. RBC count 6.4 x 10(power of 6)/uL

b

A patient with a hemoglobin (Hgb) level of 7.8 g/dL (78 g/L) has cardiac palpitations, a heart rate of 102 bpm, and an increased reticulocyte count. At this severity of anemia, what other manifestation would the nurse expect the patient to exhibit? a. Pallor b. Dyspnea c. A smooth tongue d. Sensitivity to cold

b

During physical assessment of a patient with thrombocytopenia, what would the nurse expect to find? a. Sternal tenderness b. Petechiae and purpura c. Jaundiced sclera and skin d. Tender, enlarged lymph nodes

b

If a patient with blood type O Rh+ is given AB Rh- blood, which would the nurse expect to happen? a. The patient's Rh factor will react with the RBCs of the donor blood. b. The anti-A and anti-B antibodies in the patient's blood will hemolyze the donor blood. c. The anti-A and anti-B antibodies in the donor blood will hemolyze the patient's blood. d. No adverse reaction is expected because the patient has no antibodies against the donor blood.

b

Meeting the developmental tasks of young adults with cystic fibrosis becomes a major problem primarily because a.they eventually need a lung transplant. b.they must also adapt to a chronic disease. c.any children they have will develop cystic fibrosis. d.their illness keeps them from becoming financially independent.

b

The nurse practitioner orders blood work for B.C. While reviewing the results of her complete blood count (CBC), which finding would be of greatest concern to the nurse? a. Hemoglobin of 11.4 g/dL b. Platelet count of 80,000/μL c. White cell count of 4500/μL d. Red cell count of 4.5 × 10^6/μL

b

The nurse recognizes that additional teaching is needed when the patient with asthma says a."I should exercise every day if my symptoms are controlled." b."I may use over-the-counter bronchodilator drugs occasionally if I develop chest tightness." c."I should inform my spouse about my medications and how to get help if I have a severe asthma attack." d."A diary to record my medication use, symptoms, peak expiratory flow rates, and activity levels will help in adjusting my therapy."

b

The patient has had COPD for years and his ABGs usually show hypoxia (PaO2<60 mm Hg or SaO2<88%) and hypercapnia (PaCO2>45 mm Hg). Which ABG results show movement toward respiratory acidosis and further hypoxia indicating respiratory failure? a.pH 7.35, PaO2 62 mm Hg, PaCO2 45 mm Hg b.pH 7.34, PaO2 45 mm Hg, PaCO2 65 mm Hg c.pH 7.42, PaO2 90 mm Hg, PaCO2 43 mm Hg d.pH 7.46, PaO2 92 mm Hg, PaCO2 32 mm Hg

b

To decrease the patient's sense of panic during an acute asthma attack, what is the best action for the nurse to do? a. Leave the patient alone to rest in a quiet, calm environment. b. Stay with the patient and encourage slow, pursed lip breathing. c. Reassure the patient that the attack can be controlled with treatment. d. Let the patient know that frequent monitoring is being done using measurement of vital signs and SpO2.

b

What causes the pulmonary vasoconstriction leading to the development of cor pulmonale in the patient with COPD? a.Increased viscosity of the blood b.Alveolar hypoxia and hypercapnia c.Long-term low-flow oxygen therapy d.Administration of high concentrations of oxygen

b

What is an indication of marked bronchoconstriction with air trapping and hyperinflation of the lungs in a patient with asthma? a.SaO2 of 85% b. Peak (expiratory) flow meter (PEF) rate of <200 L/min c. Forced expiratory volume in 1 second (FEV1) of 85% of predicted d. Chest x-ray showing a flattened diaphragm

b

What is characteristic of a partial rebreathing mask? a.Used for long-term O2 therapy b.Reservoir bag conserves oxygen c.Provides highest oxygen concentrations d.Most comfortable and causes the least restriction on activities

b

Which dietary modification helps to meet the nutritional needs of patients with COPD? a. Eating a high-carbohydrate, low-fat diet b.Avoiding foods that require a lot of chewing c.Preparing most foods of the diet to be eaten hot d.Drinking fluids with meals to promote digestion

b

Molecular cytogenetics and gene analysis may be done to diagnose, stage, and help to determine treatment options for various hematologic disorders. Which sites are preferred to obtain the sample for this testing (select all that apply)? a. Skin sample b. Lymph node c. Bone marrow d. Arterial blood e. Inner cheek mucosa

b c

While administering an infusion of packed RBCs, which actions can the RN delegate to unlicensed assistive personnel (UAP) (select all that apply)? a. Verify that the IV is patent. b. Obtain the blood products from the blood bank. c. Obtain vital signs before and after the first 15 minutes. d. Monitor the blood transfusion rate and adjust as needed. e. Assist the RN with checking patient identification and blood product identification data.

b c

Pulmonary rehabilitation (PR) is designed to reduce symptoms and improve the patient's quality of life. Along with improving exercise capacity, what are the anticipated results of PR (select all that apply)? a. Decreased FEV1 b. Decreased depression c. Increased oxygen need d. decreased fear of exercise e. Decreased hospitalizations

b d e

What can be auscultated in a patient with cardiac valve problems (select all that apply)? a. arterial bruit b. heart murmurs c. pulsus alternans d. third heart sound (s3) e. pericardial friction rub f. fourth heart sound (S4)

b d f

which effects result from sympathetic nervous system stimulation of b-adrenergic receptors (select all that apply)? a. vasoconstriction b. increased heart rate c. decrease heart rate d. increased rate of impulse conduction e. decreased rate of impulse conduction f. increased force of cardiac contraction

b d f

*A nurse is providing teaching for a client who is scheduled for a bone marrow biopsy of the iliac crest. Which of the following statements made by the client indicates an understanding of the teaching?* a. "The test will be performed while I am lying flat on my back." b. "I will need to stay in bed for about an hour after the test." c. "This test will determine which antibiotic I should take for the treatment." d. "I will receive general anesthesia before the test."

b. "I will need to stay in bed for about an hour after the test."

*A nurse is preparing to administer packed RBCs To a client who has a Hgb of 8 g/dL. Which of the following actions should the nurse plan to take during the first 15 minutes of the transfusion?* a. Obtain consent from the client for the transfusion b. Assess for a cute hemolytic reaction c. Explain the transfusion procedure to the client d. Obtain blood cultures specimens to send the lab

b. Assess for a cute hemolytic reaction

*A nurse is caring for a client who is receiving warfarin for anticoagulation therapy. Which of the following laboratory test results indicates to the nurse that the client needs an increase in the dosage?* a. aPTT 38 seconds b. INR 1.1 c. PT 22 seconds d. D-dimer negative

b. INR 1.1

*Which heart valve sound is best heard at the left midclavicular line at the level of the fifth ICS?* a. Aortic b. Mitral c. Tricuspid d. Pulmonic

b. Mitral Rationale: The mitral valve can be assessed by auscultation at the left midclavicular line at the fifth intercostal space (ICS).

*A P wave on an ECG represents an impulse arising at the* a. SA node and repolarizing the atria b. SA node and depolarizing the atria c. AV node and depolarizing the atria d. AV node and spreading to the bundle of His

b. SA node and depolarizing the atria Rationale: The first wave, P, begins with the firing of the sinoatrial (SA) node and represents depolarization of the fibers of the atria.

*Which subjective data related to the cardiovascular system should be obtained from the patient?* Select all that apply a. Annual income b. Smoking history c. Religious preference d. Number of pillows used to sleep e. Blood for basic laboratory studies

b. Smoking history c. Religious preference d. Number of pillows used to sleep Rationale: The health history should include assessment of tobacco use. The patient should be asked about any cultural or religious beliefs that may influence the management of the cardiovascular problem. Patients with heart failure may need to sleep with the head elevated on pillows or sleep in a chair.

*A nurse is completing an integumentary assessment of a client who has an anemia. Which of the following findings should the nurse expect?* a. Absent turgor b. Spoon shaped nails c. Shiny, hairless legs d. Yellow mucous membranes

b. Spoon shaped nails

*A nurse in the clinic receives a phone call from a client seeking information about a new prescription for erythropoietin. Which of the following information should the nurse review with the client?* a. The clients needs an erythrocyte sedimentation rate (ESR) test weekly. b. The client should have their hemoglobin checked twice a week. c. Oxygen saturation levels should be monitored. d. Folic acid production will increase.

b. The client should have their hemoglobin checked twice a week.

*An anticoagulant such as warfarin that interferes with prothrombin production will alter the clotting mechanism during* a. platelet aggregation b. activation of thrombin c. the release of tissue thromboplastin d. stimulation of factor activation complex

b. activation of thrombin

*The part of the vascular system responsible for hemostasis is the* a. thin capillary vessels b. endothelial layer of the arteries c. elastic middle layer of the veins d. smooth muscle of the arterial wall

b. endothelial layer of the arteries Rationale: The innermost lining of the arteries is the endothelium. The endothelium maintains hemostasis, promotes blood flow, and under normal conditions, inhibits blood coagulation.

*If a lymph node is palpated, what is a normal finding?* a. hard, fixed nodes b. firm, mobile nodes c. enlarged, tender nodes d. hard, nontender nodes

b. firm, mobile nodes

*A person who lives at a high altitude may normally have an increased Hgb and Hct count because* a. high altitudes cause vascular fluid loss, leading to hemoconcentration b. hypoxia caused by decreased atmospheric O2 stimulates erythropoiesis c. the function of the spleen in removing old RBCs is impaired at high altitudes d. impaired production of platelets leads to proportionally higher red cell counts

b. hypoxia caused by decreased atmospheric oxygen stimulates erythropoiesis

A 76-year-old woman has an Hgb of 7.3 g/dL (73 g/L) and is experiencing ataxia and confusion on admission to the hospital. What is a priority nursing intervention for this patient? a. Provide a darkened, quiet room. b. Have the family stay with the patient. c. Keep top bedside rails up and call bell in close reach d. Question the patient about possible causes of anemia

c

A patient is being discharged with plans for home O2 therapy provided by an O2 concentrator with an O2-conserving portable unit. In preparing the patient to use the equipment, what should the nurse teach the patient? a.The portable unit will last about 6 to 8 hours. b.The unit is strictly for portable and emergency use. c.The unit concentrates O2 from the air, providing a continuous O2 supply. d.Weekly delivery of one large cylinder of O2 will be necessary for a 7- to 10-day supply of O2.

c

A patient is being treated with chemotherapy. The nurse revises the patient's care plan based on which result? a. WBC count 4000/uL b. RBC count 4.3 x 10(power of 6)/uL c. Platelets 50,000/uL d. Hematocrit (Hct) 39%

c

A patient is receiving a drug that decreases afterload. To evaluate the patient's response to this drug, what is most important for the nurse to assess? a. Heart rate b. Lung sounds c. Blood pressure d. Jugular venous distention

c

A patient with asthma has the following arterial blood gas (ABG) results early in an acute asthma attack: pH 7.48, PaCO2 30 mm Hg, PaO2 78 mm Hg. What is the most appropriate action by the nurse? a. Prepare the patient for mechanical ventilation. b.Have the patient breathe in a paper bag to raise the PaCO2. c. Document the findings and monitor the ABGs for a trend toward alkalosis. d. Reduce the patient's oxygen flow rate to keep the PaO2 at the current level.

c

A patient's BP has not responded consistently to prescribed drugs for hypertension. The first cause of this lack of responsiveness the nurse should explore is a. progressive target organ damage. b. the possibility of drug interactions. c. the patient not adhering to therapy. d. the patient's possible use of recreational drugs.

c

An increase in which blood cell indicates an increased rate of erythropoiesis? a. Basophil b. Monocyte c. Reticulocyte d. Lymphocyte

c

The patient's lab results show a marked decrease in RBCs, WBCs, and platelets. What term should the nurse use when reporting the results to the physician? a. Hemolysis b. Leukopenia c. Pancytopenia d. Thrombocytosis

c

What does the nurse include when planning for postural drainage for the patient with COPD? a. Schedules the procedure 1 hour before and after meals b. Has the patient cough before positioning to clear the lungs c .Assesses the patient's tolerance for dependent (head-down) positions d. Ensures that percussion and vibration are performed before positioning the patient

c

What is the pathophysiologic mechanism of cystic fibrosis leading to obstructive lung disease? a. Fibrosis of mucous glands and destruction of bronchial walls b. Destruction of lung parenchyma from inflammation and scarring c. Production of secretions low in sodium chloride and therefore thickened mucus d. Increased serum levels of pancreatic enzymes that are deposited in the bronchial mucosa

c

What should the nurse include when teaching the patient with COPD about the need for physical exercise? a. All patients with COPD should be able to increase walking gradually up to 20 minutes per day. b. A bronchodilator inhaler should be used to relieve exercise-induced dyspnea immediately after exercise. c. Shortness of breath is expected during exercise but should return to baseline within 5 minutes after the exercise. d.Monitoring the heart rate before and after exercise is the best way to determine how much exercise can be tolerated.

c

When reviewing the results of an 83-year-old patient's blood tests, which finding would be of most concern to the nurse? a. Platelets 150,000/uL b. Serum iron 50 mcg/dL c. Partial thromboplastin time (PTT) 60 seconds d. Erythrocyte sedimentation rate (ESR) 35 mm in 1 hour

c

When teaching a patient about a bone marrow examination, what should the nurse explain? a. The procedure will be done under general anesthesia because it is so painful. b. The patient will not have any pain after the area at the puncture site is anesthetized. c. The patient will experience a brief, very sharp pain during aspiration of the bone marrow. d. There will be no pain during the procedure, but an ache will be present several days afterward.

c

When teaching the patient with mild asthma about the use of the peak flow meter, what should the nurse instruct the patient to do? a.Carry the flow meter with the patient at all times in case an asthma attack occurs b. Use the flow meter to check the status of the patient's asthma every time the patient takes quick-relief medication c. Follow the written asthma action plan (e.g., take quick-relief medication) if the expiratory flow rate is in the yellow zone d.Use the flow meter by emptying the lungs, closing the mouth around the mouthpiece, and inhaling through the meter as quickly as possible

c

Which breathing technique should the nurse teach the patient with moderate COPD to promote exhalation? a.Huff coughing b.Thoracic breathing c.Pursed lip breathing d.Diaphragmatic breathing

c

Which component of normal hemostasis involves the processes of protein C and protein S and plasminogen? a. Activation b. Aggregation c. Clot retraction and dissolution d. Platelet plug formation

c

The nurse determines that which blood pressure would meet the criteria for a diagnosis of stage 1 hypertension (select all that apply)? a. 134/84 mm Hg b. 138/88 mm Hg c. 144/92 mm Hg d. 156/96 mm Hg e. 182/100 mm Hg

c d

The nurse determines that the patient has stage 2 hypertension when the patient's average blood pressure is (select all that apply) a. 150/96 mm Hg. b. 155/88 mm Hg. c. 172/92 mm Hg. d. 160/110 mm Hg. e. 182/106 mm Hg

c d e

Which arteries are the major providers of cornary ciruclation (select all that apply)? a. left marginal artery b. right marginal artery c. left circumflex artery d. right coronary artery e. posterior descending artery f. left anterior descending artery

c d f

Which descriptions are characteristic of iron-deficiency anemia (select all that apply)? a. Lack of intrinsic factor b. Autoimmune-related disease c. Most common type of anemia d. Associated with chronic blood loss e. May occur with removal of the stomach f. May occur with removal of the duodenum

c d f

A 60-year-old male farmer is diagnosed with multiple myeloma. He has pain in his ribs with movement and his diagnostic studies show hypercalcemia. What nursing interventions should be implemented for this patient as the interprofessional care is being initiated? (select all that apply) a. Provide privacy b. Complete bed rest c. Adequate hydration d. Prepare for dialysis e. Assess for infection f. Encourage ambulation

c e f

Which medications are the most ef fective in improving asthma control by reducing bronchial hyperresponsiveness, blocking the late-phase reaction, and inhibiting migration of inflammatory cells (select all that apply)? a.Zileuton (Zyflo CR) b.Omalizumab (Xolair) c.Fluticasone (Flovent) d.Salmeterol (Serevent) e.Montelukast (Singulair) f.Budesonide (Pulmicort) g.Beclomethasone (Qvar) h.Methylxanthine (theophylline) i.Mometasone (Asmanex Twisthaler)

c f g i

*A nurse is providing preoperative teaching for a client who requests autologous donation in preparation for a scheduled orthopedic surgical procedure. Which of the following statements should the nurse include in the teaching?* a. "You should make an appointment to donate blood eight weeks prior to surgery." b. "If you need an autologous blood transfusion, the blood of your brother donates can be used." c. "You can donate blood each week if your hemoglobin is stable." d. "Any unused blood that is donated can be used for other clients."

c. "You can donate blood each week if your hemoglobin is stable."

*A nurses caring for a client who has idiopathic thrombocytopenic purpura (ITP). The nurse should notify the provider and report possible small-vessel clotting when which of the following is assessed?* a. Petechiae on the upper chest b. Hypotension c. Cyanotic nail beds d. Severe headache

c. Cyanotic nail beds

*A nurse is assessing a client and suspects the client is experiencing DIC. Which of the following physical findings should the nurse anticipate?* a. Bradycardia b. Hypertension c. Epistaxis d. Xerostomia

c. Epistaxis

*A nurse is teaching a client who has a new prescription for ferrous sulfate. Which of the following information should the nurse include in the teaching?* a. Stools will be dark red b. Take with a glass of milk if gastrointestinal distress occurs c. Foods high in vitamin C will promote absorption d. Take for 14 days

c. Foods high in vitamin C will promote absorption

*A nurse is monitoring a client who began receiving a unit of packed RBCs 10 minutes ago. Which of the following findings should the nurse identify as a indication of febrile transfusion reaction?* Select all that apply a. Temperature change from 37°C pre-transfusion to 37.2°C b. Current blood pressure 178/90 mm Hg c. Heart rate change from 88/minute pre-transfusion to 120/minute d. Client report of itching e. Client appears flushed

c. Heart rate change from 88/minute pre-transfusion to 120/minute e. Client appears flushed

*You are taking care of a male patient who has the following laboratory values from his CBC: WBC 6.6 x 10 3/uL, Hgb 13.4 g/dL, Hct 40%, platelets 50 x 10 3/uL. What are you most concerned about?* a. The patient is neutropenic. b. The patient has an infection. c. There is an increased risk for bleeding. d. Fall risk precautions are needed due to anemia.

c. There is an increased risk for bleeding.

*An unexpected finding in the assessment of an 79-year-old patient is* a. a narrowed pulse pressure b. diminished carotid artery pulses c. difficulty in isolating the apical pulse d. an increased heart rate in response to stress

c. difficulty in isolating the apical pulse Rationale: Myocardial hypertrophy and the downward displacement of the heart in an older adult may cause difficulty in isolating the apical pulse.

*While assessing the lymph nodes, the nurse should* a. apply gentle, firm pressure to deep lymph nodes b. palpate the deep cervical and supraclavicular nodes last c. lightly palpate superficial lymph nodes with the pads of the fingers d. use the tips of the second, third and fourth fingers to apply deep palpation

c. lightly palpate superficial lymph nodes with the pads of the fingers

*A patient with a tricuspid valve disorder will have impaired blood flow between the* a. vena cava and right atrium b. left atrium and left ventricle c. right atrium and right ventricle d. right ventricle and pulmonary artery

c. right atrium and right ventricle Rationale: The tricuspid valve is located between the right atrium and right ventricle.

A lymph node biopsy is most often performed to diagnose a. leukemia. b. cause of lymphedema. c. hemorrhagic tendencies. d. neoplastic cells in lymph nodes.

d

A patient returns to the cardiac observation area following a cardiac catheterization with coronary angiography. Which assessment would require immediate action by the nurse? a. Pedal pulses are 2+ bilaterally. b. Apical pulse is 54 beats/minute. c. Mean arterial pressure is 72 mm Hg. d. ST-segment elevation develops on the ECG.

d

A patient's blood pressure has not responded to the prescribed drugs for hypertension. Which of the following should the nurse assess first? a. Potential for drug interactions b. Progressive target organ damage c. Possible use of recreational drugs d. Patient's adherence to drug therapy

d

During an acute exacerbation of mild COPD, the patient is severely short of breath and the nurse identifies a nursing diagnosis of ineffective breathing pattern related to alveolar hypoventilation and anxiety. What is the best action by the nurse? a.Prepare and administer routine bronchodilator medications. b.Perform chest physiotherapy to promote removal of secretions. c.Administer oxygen at 5L/min until the shortness of breath is relieved. d.Position the patient upright with the elbows resting on the over-the-bed table.

d

In addition to smoking cessation, what treatment is included for COPD to slow the progression of the disease? a.Use of bronchodilator drugs b.Use of inhaled corticosteroids c.Lung volume-reduction surgery d.Prevention of respiratory tract infections

d

In an adult patient with bronchiectasis, what is a nursing assessment likely to reveal? a.Chest trauma b.Childhood asthma c.Smoking or oral tobacco use d.Recurrent lower respiratory tract infections

d

In planning care for the patient with bronchiectasis, which nursing intervention should the nurse include? a.Relieve or reduce pain b.Prevent paroxysmal coughing c.Prevent spread of the disease to others d.Promote drainage and removal of mucus

d

The husband of a patient with severe COPD tells the nurse that he and his wife have not had any sexual activity since she was diagnosed with COPD because she becomes too short of breath. What is the nurse's best response? a."You need to discuss your feelings and needs with your wife so she knows what you expect of her." b."There are other ways to maintain intimacy besides sexual intercourse that will not make her short of breath." c."You should explore other ways to meet your sexual needs since your wife is no longer capable of sexual activity." d."Would you like for me to talk to you and your wife about some modifications that can be made to maintain sexual activity?"

d

Tobacco smoke causes defects in multiple areas of the respiratory system. What is a long-term effect of smoking? a.Bronchospasm and hoarseness b.Decreased mucus secretions and cough c.Increased function of alveolar macrophages d.Increased risk of infection and hyperplasia of mucous glands

d

What is the most effective treatment for cystic fibrosis? a.Heart-lung transplant b.Administration of prophylactic antibiotics c.Administration of nebulized bronchodilators d.Vigorous and consistent chest physiotherapy

d

What is the primary principle involved in the various airway clearance devices used for mobilizing secretions? a.Vibration b.Inhalation therapy c.Chest physiotherapy d.Positive expiratory pressure

d

When teaching the patient about going from a metered-dose inhaler (MDI) to a dry powder inhaler (DPI), which statement by the patient shows the nurse that the patient needs more teaching? a."I do not need to use the spacer like I used to." b."I will hold my breath for 10 seconds or longer if I can." c."I will not shake this inhaler like I did with my old inhaler." d."I will store it in the bathroom so I will be able to clean it when I need to."

d

Which medication is a long acting beta adrenergic agonist and dry powder inhaler (DPI) that is used only for COPD? a.Roflumilast (Daliresp) b.Salmeterol (Serevent) c.Ipratropium (Atrovent HFA) d.Indacterol (Arcapta Neohaler)

d

Which medication should the nurse anticipate being used first in the emergency department for relief of severe respiratory distress related to asthma? a. Prednisone orally b. Tiotropium inhaler c. Fluticasone inhaler d. Albuterol nebulizer

d

Which obstructive pulmonary disease would a 30-year-old white female patient with a parent with the disease bemost likely to be diagnosed with? a.COPD b.Asthma c.Cystic fibrosis d.α1-Antitrypsin (AAT) deficiency

d

While assisting a patient with intermittent asthma to identify specific triggers of asthma, what should the nurse explain? a. Food and drug allergies do not manifest in respiratory symptoms. b.Exercise-induced asthma is seen only in individuals with sensitivity to cold air. c.Asthma attacks are psychogenic in origin and can be controlled with relaxation techniques. d.Viral upper respiratory infections are a common precipitating factor in acute asthma attacks

d

what is a significant finding in the health history of a patient during an assessment of the cardiovascular system? a. metastatic cancer b. calcium supplementation c. frequent viral pharyngitis d. frequent use of recreational drugs

d

*A nurse is teaching a newly licensed nurse about heparin-induced thrombocytopenia. Which of the following risk factors for this disorder should the nurse include in the teaching?* a. Warfarin therapy for atrial fibrillation b. Placental abruption c. Systemic lupus Erythematosus d. Heparin therapy for deep vein thrombosis

d. Heparin therapy for deep vein thrombosis

*A nurse is caring for a client who has suspected anemia. Which of the following laboratory test results should the nurse expect?* a. Iron 90 mcg/dL b. RBC 6.5 mill/uL c. WBC 4800 mm3 d. Hgb 10 g/dL

d. Hgb 10 g/dL

*A nurse preceptor is observing a newly licensed nurse on the unit who is preparing to administer a blood transfusion to an older adult client. Which of the following actions by the newly licensed nurse indicates an understanding of the procedure?* a. Inserts and 18 gauge IV catheter in the client b. Verifies blood compatibility and expiration date of the blood with an assistive personnel (AP) c. Administers dextrose 5% in 0.9% sodium chloride IV with the transfusion d. Obtains vital signs every 15 minutes throughout the procedure

d. Obtains vital signs every 15 minutes throughout the procedure

*Cancer arising from granulocytic cells in the bone marrow will have the primary effect of causing* a. risk for hemorrhage b. altered oxygenation c. decreased production of antibodies d. decreased phagocytosis of bacteria

d. decreased phagocytosis of bacteria


Ensembles d'études connexes

Pre-Calc Ch. 10-12 Unit Circle/Trig EQUATIONS

View Set

ap chem ap multiple choice chapter 10

View Set

Select the Correct Answer Exercise 10.5: Identify the drain/tube/catheter as either passive (P) or active (A).

View Set

Chapter 3 - The Basics Health - Rebecca J Donatelle

View Set

Small Business Entrepreneurship A

View Set